Sail E0 Webinar

Reasoning Aptitude

STATEMENT AND CONCLUSION MCQs

Statement Conclusions And Inferences

Total Questions : 618 | Page 31 of 62 pages
Question 301.

In each question below is given a statement followed by two conclusions numbered I and II. You have to assume everything in the statement to be true, then consider the two conclusions together and decide which of them logically follows beyond a reasonable doubt from the information given in the statement.


Give answer:



  • (A) If only conclusion I follows

  • (B) If only conclusion II follows

  • (C) If either I or II follows

  • (D) If neither I nor II follows and

  • (E) If both I and II follow.




Statements: I know nothing except the fact of my ignorance.


Conclusions:



  1. Writer's knowledge is very poor.

  2. The world of knowledge is too vast to be explored by a single person.

  1.    Only conclusion I follows
  2.    Only conclusion II follows
  3.    Either I or II follows
  4.    Neither I nor II follows
  5.    Both I and II follow
 Discuss Question
Answer: Option B. -> Only conclusion II follows
The statement is a symbolic one and only II correctly explains it.
Question 302.

In each question below is given a statement followed by two conclusions numbered I and II. You have to assume everything in the statement to be true, then consider the two conclusions together and decide which of them logically follows beyond a reasonable doubt from the information given in the statement.


Give answer:



  • (A) If only conclusion I follows

  • (B) If only conclusion II follows

  • (C) If either I or II follows

  • (D) If neither I nor II follows and

  • (E) If both I and II follow.




Statements: Company X has a record of manufacturing cameras of quality and the latest design so that you do not spoil even a single shot irrespective of the weather conditions.


Conclusions:



  1. No other company except X is reputed in the camera industry.

  2. Anyone can take an acceptable shot with camera X.

  1.    Only conclusion I follows
  2.    Only conclusion II follows
  3.    Either I or II follows
  4.    Neither I nor II follows
  5.    Both I and II follow
 Discuss Question
Answer: Option B. -> Only conclusion II follows
Clearly, the statement talks of Company X only and no other company. So, I does not follow. Also, it is mentioned that one can take a good shot even in bad weather conditions with a camera of Company X. So, II follows.
Question 303. Statement:Excess usage of pesticides has resulted in pests developing immunity. Thereby farmers are suffering from heavy loss in their crops as a result of these resurgent pests.Courses of Action:I. Farmers should stop using pesticides.II. Research must be done in producing hybrid varieties which are very strong and cannot be destroyed by pests.III. Farmers should stop growing crops for some time.
  1.    Only II follows
  2.    Only III follows
  3.    Only I follows
  4.    None follows
  5.    None of these
 Discuss Question
Answer: Option A. -> Only II follows


Course of action I and III are negative course of action. Here the problem is regarding pests. As, the course of action  II says about the research which will help to produce hybrid varieties which are strong enough to fight against pests, hence it follows.


Question 304. Statement:Cutting down of forests is a threat to the wild life. Most of the species of animals are on the verge of extinction.Courses of Action:I. The species of animals, which are on the verge of extinction, must be protected by creating wild life sanctuaries which contain flora that defines the native habitat of the endangered animals.II. To the maximum extent possible we should stop deforestation.III. Growing urban forests to compensate for deforestation. 
  1.    Only II follows
  2.    Only III follows
  3.    Only I and II follow
  4.    Only I and follows
  5.    Either I or II follows
 Discuss Question
Answer: Option C. -> Only I and II follow


Here the problem is extinction of various species of animals, so providing them their native habitat is the proper solution for the problem.II is a proper course of action, as stopping deforestation can solve the problem to some extent.III is not a proper course of action as urban forest cannot be the habitat for the wild animals.


Question 305. Statement:
Our neighbouring country is manufacturing nuclear weapons. It's a threat to our country.Courses of Action:I. We should disclose what arms we have so that the neighbouring country will be deterred.II. We should conduct a series of nuclear tests.III. We should be alert by all means.
  1.    Only I and II follow
  2.    Only III follows
  3.    Only II follows
  4.    Only I follows
  5.    All the three follow
 Discuss Question
Answer: Option B. -> Only III follows


Disclosing all arms to the enemy country is not a proper course of action to avoid threat. Hence, I does not follow. Course of action II will create a new problem, as that will led to arms race. Hence II is not a proper course of action. III is a proper course of action as being alert by all means is always the solution of a threate.


Question 306. Statement:Out of every hundred people who are blind, twenty people go blind because of the deficiency of Vitamin 'A'. But this fact is not known to illiterates and it is neglected by the educated.Courses of Action:I. Literacy programmes must be launched to educate people about this fact.II. Educated people must be punished.III. As television is the main media which reaches 80% of the population, the government should make a small feature film, explaining the significance of various vitamins.
  1.    Only I follows
  2.    Only I and III follow
  3.    Either II or III follows
  4.    All follow
  5.    None follows.
 Discuss Question
Answer: Option A. -> Only I follows


As people can be educated regarding vitamins.This a valid course of action.II is a negative course of action.III is based on the assumption that such types of programmes were not there before.So, we are not certain about the result.


Question 307. Statement:
Residents from Model Colony coming under the north-ward of city 'X' have complained to the ward officer that for the last three days the tap water in the ward has been contaminated and no action has been initiated by the municipal staff.Course of Action:I. The ward officer of the north-ward should initiate an action against residents  who have lodged complaints against the municipal staff.II. The ward officer should ask the ward engineer to check water installations and get samples of water tested from laboratories.III. People belonging to the north-ward of the city should go for their own borewells.
  1.    Only I follows
  2.    Only II follows
  3.    Only II and III follow
  4.    None follows
  5.    None of these.
 Discuss Question
Answer: Option D. -> None follows


Here the problem is with the municipal staff who did not take any initiate though the water got contaminated.I is a negative course of action, hence does not follow.Initiative must be taken by municipal staff, not by word officers, hence II does not follow.III is not proper course of action because it is not practically feasible for every family to go with their own bore wells. Hence, none follows.


Question 308. Should the director with political influence be preferred to the director with vast experience for the post of managing director?Arguments:I. Yes, now  a days, business becomes successful by exercising influence rather than using strategies.II. No, strategies developed by an experienced businessman makes a business definitely successful.
  1.    if only argument I is strong.
  2.    if only argument II is strong.
  3.    if either I or II is strong.
  4.    if neither I nor II is strong.
  5.    if both I and II are strong.
 Discuss Question
Answer: Option B. -> if only argument II is strong.


"Influence" may be interpreted in more than one way. Statement I says that "influences" is better than "strategies" but it is not referring directly to "political influence" which is what is given in the passage. Hence, this is a weak argument. "Experience" is to be constructed in only one way, that is experience in developing strategies which is a part of the job of a director. Hence this is a strong argument.


Question 309. Should foreign print media be allowed to enter the Indian market?Arguments:I. Yes, Indian media networks are not so effective as foreign media networks area.II. No, valuable, confidential government information would be put at stake.
  1.    if only argument I is strong.
  2.    if only argument II is strong.
  3.    if either I or II is strong.
  4.    if neither I nor II is strong.
  5.    if both I and II are strong.
 Discuss Question
Answer: Option E. -> if both I and II are strong.


Ineffectiveness of the existing players is an important point to be considered while examining new proposals. Hence, statement I is a strong argument. If true, even statement II which says that national security could be put in jeopardy, would provide a valid and strong argument against foreign media networks being allowed in India.


Question 310. Should children be allowed to use internet in internet kiosks.Arguments:I. Yes, all the internet kiosks are using filter-software's, which do not allow adult software to be downloaded.II. No, internet is a communication medium, which transmits information and does not distinguish between a child and a grown-up.
  1.    if only argument I is strong.
  2.    if only argument II is strong.
  3.    if either I or II is strong.
  4.    if neither I nor II is strong.
  5.    if both I and II are strong.
 Discuss Question
Answer: Option A. -> if only argument I is strong.


One important reason to stop children from using kiosks is audit material available on the net. Hence, statement I is a strong argument. Statement II gives an irrelevant reason as its argument and is very weak.


Latest Videos

Latest Test Papers